Evidence Quiz Questions

¡Supera tus tareas y exámenes ahora con Quizwiz!

Sarah is charged with a complex financial fraud, in which she allegedly orchestrated the sale of bogus securities while working at a major investment banking firm. The firm's aging CEO testifies before a grand jury how Sarah arranged sales of securities that actually did not exist, and how the firm uncovered this fraud. By Sarah's trial, the CEO has retired and is now quite elderly. He testifies to Sarah's fraud scheme but acknowledges he no longer recalls many of the details as to how it was identified. Sarah's lawyer cross examines the retired CEO asking how he can be so sure about her fraud if he no longer recalls how it was identified. The retired CEO testifies that he simply doesn't remember. After the retired CEO is cross examined, the prosecution offers the retired CEO's grand jury testimony to show that he previously described the fraud the same way he does now. Sarah's lawyer objects that the retired CEO's entire testimony, on direct and redirect, is inadmissible under the confrontation clause because he cannot be meaningfully confronted. How should the judge rule? A. Exclude the testimony because the CEO's inability to recall anything about how the fraud was discovered makes confronting his accusation functionally impossible. B. Admit the testimony because the CEO's statements were not testimonial as he was describing primarily historical events. C. Exclude the testimony because the CEO was not previously subject to cross examination before the grand jury. D. Admit the testimony because the CEO was subject to confrontation during his cross examination even if it was ineffective due to his lack of memory.

D D is correct. The confrontation clause guarantees only the opportunity confront one's accusers, even if that opportunity is not particularly successful. US v. Owens. A is wrong because the difficulty of confronting a witness with failing memory concerning the witness's prior statements does not deprive the defendant of the opportunity for cross examination. US v. Owens. B is wrong because the CEO's statements were testimonial. Both his direct examination and his statements before the grand jury were in fact testimony, and they were offered by the prosecution as accusations of Sarah. Recounting historical events as part of building a case by the state is one of the characteristic features of testimonial statements. Hammond v. Indiana. C is wrong because even though the CEO was not subject to cross-examination before the grand jury, he is subject to cross examination now, which satisfies the confrontation clause. US v. Owens.

Harry is charged with robbing a bank and offers Carol as a witness. Carol would testify that she was in a bar the week after the bank robbery where she met Pete - for the first time - who told her without prompting "in the strictest confidence, because he didn't have long to live" that he had actually committed the bank robbery. Pete and Carol spoke for the rest of the evening and exchanged phone numbers, but shortly thereafter Pete died. The prosecution objects that Carol's testimony is inadmissible hearsay. How should the judge rule? A. Admit the testimony as a statement under the belief of imminent death. B. Exclude the testimony as a statement of a coconspirator. C. Admit the testimony as a statement against interest. D. Exclude the testimony because it lacks corroborating circumstances that clearly indicate its trustworthiness.

D D is the correct answer. Although the declarant is unavailable through death, and the statement is one a reasonable person would have made only if the declarant believed it to be true because it would expose the declarant to criminal liability, because it is offered in a criminal case and exposes the declarant to criminal liability there must be corroborating circumstances that clearly indicate its trustworthiness. There are none - and there are several circumstances suggesting it is not trustworthy, namely that the statement is made to a stranger in a bar unprompted. While providing a phone number could be a means of identifying oneself, which might add some trustworthiness, there are other reasons for exchanging phone numbers than standing behind this statement. Answer A is wrong because the statement neither concerns the cause of Pete's imminent death nor is being offered in a homicide or civil case. Answer B is wrong because there is no evidence Pete is a coconspirator or that the statement was made during the conspiracy, and it clearly did not further the conspiracy. Answer C is wrong because it lacks corroborating circumstances that clearly indicate its trustworthiness, required under 804(b)(3)(B) when such a statement is offered in a criminal case and tends to expose the declarant to criminal liability.

Derek is charged with the murder of Vinnie. At trial, the prosecution calls Vinnie's neighbor Sam, to testify. Sam testifies that about an hour before Vinnie's disappearance Sam's 8-year-old son Jimmy calmly stated, "Dad, there goes Vinnie in that car with Derek and they are driving away." Derek objects that Sam's testimony is hearsay. How should the judge rule? A. Exclude the testimony because too much time has elapsed between what Jimmy saw and Vinnie's disappearance. b. Exclude the testimony because Jimmy was calm when he made the statement. C. Admit the testimony because, given Jimmy's age, a judge would determine him unavailable to testify. D. Admit the testimony because Jimmy's statement is a present sense impression.

D The correct answer is D. Here the son's statement is being made simultaneously with the event he is describing. Answer choice A is wrong because the relevant time for a present sense impression is the time between the matter observed and the statement. Answer choice B is wrong because the present sense impression exception does not require the declarant to be under any stress when making the statement. Answer choice C is wrong because availability/Unavailability of the declarant is immaterial under FRE 803(1).

In a criminal case Defendant is charged with murder for beating a woman to death in the wooded area of a local park. At trial the prosecution wants to offer a 911 recording of an eyewitness who reported seeing a man matching the defendant's description beating a woman in the same wooded area of the park on the night the murder. Which of the following is a correct statement of the law regarding the 911 recording? A. Statements made by those reporting an incident on 911 recordings are admissible as hearsay exceptions either as excited utterances or present sense impressions. B. Statements made by those reporting an incident on 911 recordings are not hearsay. C. If the statements made on 911 recordings meet the excited utterance or present sense impression exceptions to the hearsay rule, the statements will not violate the Confrontation Clause. D. Even if the statements made on 911 recordings satisfy a hearsay exception, a separate Confrontation Clause analysis must also be done if the statement is being offered against the defendant in a criminal case.

D The correct answer is D. Statements that may satisfy a hearsay exception must also be analyzed to see if the statement violates the defendant's right in a criminal case to confront the witnesses against the defendant. Thus, answer choice C is wrong. Answer choice A is wrong because the statement is too broad. Not all 911 calls fit into one of the exceptions. Answer choice B is wrong because 911 recordings are frequently hearsay statements.

Bill sues Sarah for breach of contract on January 15, 2009. At trial, in November 2009, Sarah testifies that she and Bill had an oral understanding that the terms of the contract were meant to be flexible (Bill denies there was such an understanding). Bill seeks to cross examine Sarah as follows: "Weren't you convicted in December 1999 for Driving under the Influence (DUI), for which you received a suspended sentence of two years' incarceration and two years' probation?" Assume that Bill has a certified copy of the judgment for the 1999 DUI. Sarah objects that this is inadmissible. How should the judge rule? A. Sustain the objection because this is a civil case. B. Sustain the objection because Sarah was never incarcerated. C. Overrule the objection because the DUI carried a sufficiently severe penalty and under the circumstances its probative value substantially outweighs its prejudicial effect. D. Overrule the objection because the DUI carried a sufficiently severe penalty and under the circumstances its probative value is not substantially outweighed by the risk of unfair prejudice.

D The correct answer is D. The 1999 DUI is admissible under FRE 609(a)(1)(A) because it was punishable by over a year's imprisonment, it satisfies the time bar in 609(b) (note the conviction occurred in December 1999 and trial is in November 2009, and the current date is the latest possible end date for the time bar), and there is nothing to suggest that it poses a risk of unfair prejudice that substantially outweighs its probative value (i.e., it satisfies FRE 403). Answer A is incorrect. That this is a civil case is not relevant, except that the test for FRE 609(a)(1) convictions is not the special reverse 403 test but only the standard 403 test for convictions that otherwise satisfy FRE 609. Answer B is incorrect. That Sarah was never incarcerated doesn't matter for 609(a)(1); all that matters is the maximum possible punishment, regardless of the punishment received. Answer C is incorrect. Because the DUI is under ten years old, there is no need for the special test of FRE 609(b)(1) (i.e., that its probative value must outweigh its prejudicial effect).

Defendant is being sued for medical malpractice. Defendant calls an expert witness to testify that the treatment Defendant provided was within the standard of care. Plaintiff wants to question the expert witness about the fact that she was hired by Defendant's insurance company to serve as an expert in this case. Defendant objects under Rule 411. How should the judge rule? A. Allow the question because it establishes the potential bias of the expert witness. B. Prohibit the question because evidence of liability insurance is inadmissible. C. Prohibit the question because the payment of expert witnesses is not a relevant issue in this case. D. Allow the question because the fact finder should know if Defendant has malpractice insurance.

A

Defendant is on trial for armed robbery. The prosecution's first witness is the owner of a local clothing store. Store Owner will testify that Defendant purchased a dark green "Member's Only" jacket at that store two months before the alleged robbery. Later in the trial prosecution intends to call an eyewitness to testify that the person she saw committing the armed robbery was wearing a dark green "Member's Only" jacket. Defendant objects to Store Owner's testimony. How should the judge rule? A. Admit the evidence. B. Exclude the evidence because it is irrelevant. C. Require the prosecution to first call the eyewitness to testify about what she observed the alleged robber wearing at the time of the alleged crime. D. Admit the evidence and instruct the jury that they must consider it in determining the defendant's possible guilt.

A

Jane sues American Motors, which manufactured her car, claiming breach of the implied warranty of merchantability (that the car is suitable for its intended use) because the car's engine failed after two years and 20,000 miles. American Motors defends that the engine failed because Jane did not follow its recommendation to change the oil every six months or 5,000 miles. Jane offers testimony from her friend John that he bought a car at the same time as she did, although made by a different manufacturer, and his car's engine failed after one year and 10,000 miles. American Motors objects that this evidence is not relevant. With respect only to the issue of relevance, how should the judge rule? A. The evidence is not relevant because John's car was not made by the same manufacturer as Jane's car. B. This evidence is not relevant as John is Jane's friend, thus he has a bias. C. The evidence is not relevant because John's car failed after only a year and 10,000 miles, so it is not comparable to the failure of Jane's car. D. The evidence is relevant although it may have relatively low probative value.

A

Karen is arrested for OUI and police obtain a court order to draw Karen's blood for a blood-alcohol concentration analysis (BAC). They take her to the hospital where a nurse draws a sample of her blood. The blood is analyzed by a chemist at the police crime lab who operates a gas chromatography machine that produces a report of a sample's BAC. The report shows Karen has 0.10% BAC, or 0.1 gram of alcohol per 100 milliliters of blood, which exceeds the limit of 0.08%, above which someone is presumed to be under the influence of alcohol. At Karen's OUI trial, because the chemist who ran the test is on extended sick leave, the prosecutor offers the report through a senior chemist at the lab who did not run the test but who has 25 years' experience operating the gas chromatography machines, who has performed thousands of such tests and who trains other chemists in the machine's operation. Karen objects that admitting the report violates the confrontation clause. Which is the judge's most likely ruling? A. Exclude the report because cross examination of the senior chemist concerning the operation of the machine and conducting the test does not satisfy the confrontation clause. B. Admit the report because it is a record of a regularly conducted activity, and the senior chemist can testify that the report was made by someone with knowledge, that the report is kept in the course of a regularly conducted activity (the lab) and that it is the regular practice of the lab to make such reports, given his experience in the lab and with the machines. C. Exclude the report because it was not produced in response to an ongoing emergency. D. Admit the report because the opportunity to cross examine the senior chemist concerning the operation of the machine and conducting the test satisfies the confrontation clause.

A Answer A is correct. The admission of the report violates the confrontation clause because the report is testimonial and cross examination of the senior chemist as a surrogate does not qualify as confrontation. Bullcoming v. New Mexico, 131 S.Ct. 2705 (2011).

A company discovers significant parts of its inventory are missing from its warehouse, despite no record it was sold. A company lawyer investigates and interviews the inventory control officer, who signs a statement admitting that "sometimes when stock didn't sell, I would look the other way when inventory workers would steal some of it. I realize my job was to ensure this didn't happen, and I'm sorry I allowed it to happen." The company then fires the inventory control officer. The company fires the inventory workers and sues them to recover the value of the items they stole. By the time of trial, the former inventory control officer has died, and the company offers his statement as proof that the defendant former workers stole items from the company's inventory. The defendants object that the statement is inadmissible hearsay. Which best describes how the judge will likely rule? A. The statement is admissible as a statement against interest. B. The statement is not admissible because it was not made under oath. C. The statement is not admissible because it is not supported by corroborating circumstances that clearly indicate its trustworthiness. D. The statement is admissible as a record of a regularly conducted activity.

A Answer choice A is correct. The statement is admissible as a statement against interest. 804(b)(3). The statement implicates the declarant (the inventory control officer) as having allowed people to steal from the company, which is so far contrary to his pecuniary interest (he lost his job) that a reasonable person in his situation would not have said it were it not true. Answer choice B is wrong. The party against whom the statement is offered need not have a prior chance to examine the declarant; that is for 804(b)(1) (former testimony). Even if made under oath, this would not permit the statement's admission as former testimony because the party against whom it is offered never had an opportunity to develop it by examination. Answer choice C is wrong. As this is a civil case, there is no confrontation issue and no need for corroborating circumstances clearly indicating its trustworthiness. Answer choice D is wrong. None of the requirements for that exception apply to these facts.

John is sued for negligence arising from a car accident and testifies at trial as follows: "I stopped my car at a red light at an intersection. I was behind a large truck that was blocking my view of the traffic light. When the truck started to drive forward, I assumed that meant that the light had turned green, so I started to drive through the intersection. I was immediately hit by the plaintiff's car, which drove right into me." The plaintiff objects to John's testimony on hearsay grounds, claiming that the truck driver's decision to drive through the intersection is a statement under FRE 801(a). The court disagrees, finding that the truck driver's non-verbal conduct is not hearsay. The best explanation for the court's ruling is that: A. The truck driver did not intend to communicate anything by driving through the intersection. B. Non-verbal conduct cannot be an out-of-court statement offered for the truth of the matter asserted. C. There is no risk that the truck driver misperceived the color of the traffic light before entering the traffic intersection. D. The plaintiff can call the truck driver as a witness to explain why the truck driver drove through the intersection.

A The correct answer is A. Non-verbal conduct that is not intended as an assertion is not a "statement" under FRE 801(a). The rationale for this rule is that a person who acts without the intent to communicate cannot be insincere, and therefore the risk that the person is lying -- which is a core concern of the hearsay rule -- does not exist. Answer B is incorrect because nonverbal conduct that is intended to communicate can be hearsay if offered for its truth. Answer C is incorrect because the truck driver may have misperceived whether the traffic light had turned green. The hearsay rule is not concerned with this type of misperception under these circumstances. Answer D is incorrect because determining whether the truck driver's conduct is a statement under FRE 801(a) does not depend on whether the truck driver can be called as a witness at the trial.

A car driven by Don crashed into a car driven by Peter. George was a passenger in Peter's car. Peter sues Don and at trial he calls Wendy Witness to testify as to what she saw. Separately, George also sues Don because of injuries he sustained in the car accident, and the trial of George's suit occurs after the trial of Peter's. Shortly after George filed his lawsuit, and without warning, Wendy moved to the Antarctic to continue her work with penguins and the judge determines that she was unavailable to testify after the subpoena was returned undelivered. George offers Wendy's testimony from the Peter v. Don trial. Peter objects that this is hearsay. How should the judge rule? A. This testimony is admissible as former testimony. B. This testimony is inadmissible because Peter is not a predecessor in interest to George. C. This testimony is inadmissible because Wendy is not unavailable. D. This testimony is admissible because Wendy's testimony is against Don's pecuniary interest.

A The correct answer is A. This comes in under 804(b)(1). Don was the defendant in the earlier proceeding and would have had the opportunity and a similar motive to cross-examine Wendy in the first trial. Answer choice B is wrong because the focus is not on the relationship between George and Peter, but on the defendant Don who was a defendant in both trials. Answer choice C is wrong because the witness is unavailable under 804(a)(5). Answer choice D is wrong. In order for a statement against interest to be admissible as a hearsay exception, the statement must be against the declarant's interests, not someone else's interest.

Stan, who has no criminal record, was talked into committing a burglary by his best friend Ollie. The crime was supposed to be simple: they would enter the back entrance of the corner deli, steal money out of the cash register, and flee. All went according to plan until they were two blocks from the deli, where Stan and Ollie spotted a police officer walking down the street. As the officer approached Stan, he started to shake violently from nerves. Seeing Stan's nervous behavior, the officer asked him for identification, at which point Stan blurted out, "I'm sorry, I'll never commit another crime!" At trial, the prosecution calls the police officer who testifies that he asked Stan for identification because he appeared so nervous. The defense objects, claiming that Stan's nonverbal behavior is hearsay. How should the court rule? A. Overrule the objection because Stan's nervous conduct is not hearsay. B. Overrule the objection because Stan's subsequent statement that he would "never commit another crime" is sufficient to convict Stan of the burglary. C. Sustain the objection because Stan's conduct was an out-of-court nonverbal assertion offered to prove the matter asserted. D. Sustain the objection because the officer does not have personal knowledge of why Stan was nervous.

A The correct answer is A. To be hearsay, there must be an out-of-court statement, which can include nonverbal conduct that is intended as an assertion. Here, there is no indication that Stan had any intent to communicate anything to anyone by his nervous behavior. Rather, the facts indicate that Stan, as a first time criminal, could not control his nervous behavior when he was approached by the police officer. Therefore, Stan's nonverbal conduct is not a statement under FRE 801(a) and proof regarding his nervousness is not barred by the hearsay rule. Answer B is incorrect because the strength of the other evidence against Stan is irrelevant to determining whether the court should admit evidence of Stan's nervous behavior. Answer C is wrong because Stan's conduct is not an assertion. Answer D is wrong because the officer has personal knowledge that Stan appeared nervous when the officer approached, which is what the prosecutor seeks to prove through the officer's testimony. (Note: as we will see, this is all that is required by FRE 602, which we will study later in the semester).

Alex and Chris leave their four-year-old son in the care of their babysitter, Sam, for an evening. The four-year-old goes to sleep, and Sam spends a quiet evening with Alex and Chris's beagle - who without provocation bites Sam's hand, requiring several stiches to close the wounds. Alex takes Sam to the emergency room and pays the $500 bill for Sam's care. Horrified at Sam's injuries, Alex and Chris write Sam a note saying "We are terribly sorry about the beagle's behavior, we've never really been able to train her, and she's always been unpredictable. We'll pay any other medical expenses from the injuries." Sam sues Alex and Chris for injuries caused by the beagle, and claims they were negligent to leave the dog alone with the babysitter. Sam seeks to introduce their note to show their liability for injuries caused by the beagle. Alex and Chris object that the note is categorically inadmissible. How should the court rule? A. The note is admissible to show that Alex and Chris felt responsible for having caused Sam's injuries, but it is not admissible to show liability for Sam's injuries. B. The first sentence of the note is admissible to show lack of care, but the second sentence of the note is categorically inadmissible. C. The second sentence of the note is admissible to show consciousness of guilt, but the first sentence of the note is categorically inadmissible. D. The note is categorically excluded because it is an offer to pay medical expenses.

B

In a conversation with his Landlord shortly after renting an apartment, Renter tells Landlord that the apartment is "great." The next month Renter tells landlord that he is short on money and can't pay the entire $1000 rent. He gives the landlord $700 and says, "here is $700; I know I owe you $1000, and I will pay you the rest when I can." Later Landlord sues Renter for the $300 balance. At trial, Renter claims he withheld $300 because the heat was not working in the apartment. Landlord now wants to testify about the statement Renter had made to him earlier about the apartment being "great." Renter objects. How should the judge rule? A. Sustain the objection because statements made during settlement offers are not admissible to prove the validity of a claim. B. Overrule the objection because Renter's statements were not a part of settlement negotiations. C. Sustain the objection because Landlord is not an unbiased witness. D. Overrule the objection because Renter would not be entitled to withhold a portion of the rent under these facts.

B

Jane sues American Motors, which manufactured her car, claiming breach of the implied warranty of merchantability (that the car is suitable for its intended use) because the car's engine failed after two years and 20,000 miles. American Motors defends that the engine failed because Jane did not follow its recommendation to change the oil every six months or 5,000 miles. Jane offers the testimony of an automotive engineer that American Motors' engines are poorly designed and routinely fail after two years or 25,000 miles. American Motors objects that this evidence is not relevant. With respect only to the issue of relevance, how should the judge rule? A. The engineer's testimony is not relevant because Jane's claim is for a breach of the warranty of merchantability rather than a defect in the design. B. The engineer's testimony is relevant because it makes more likely that the engine failed due to a defect in the design. C. The engineer's testimony is not relevant because the car failed at 20,000 miles rather than 25,000 miles. D. The engineer's testimony is relevant only if the engineer can testify that American Motors' engines have failed as early as 20,000 miles.

B

Jerzy brought a lawsuit against Massachusetts General Hospital, claiming that the surgeons who removed a blood clot from his artery committed malpractice that left him severely injured. At trial, the hospital seeks to admit into evidence a toxicology report revealing that Jerzy failed to disclose to his doctors that shortly before the surgery he had consumed marijuana, which can interfere with anesthesia and cause complications during surgery. Marijuana is legal to consume in Massachusetts. The hospital claims that the toxicology report is necessary to demonstrate the cause of Jerzy's injuries. Jerzy's lawyer objects under Rule 403. How should the court rule? A. Sustain the objection under Rule 403. B. Overrule the objection and admit the evidence. C. Overrule the objection and admit the evidence, but with a limiting instruction that the jury can consider the evidence only for determining whether Jerzy has a propensity for drug use. D. Sustain the objection because Jerzy's marijuana use is not relevant.

B

Karen is charged with a crime that occurred on March 15, 2011, and at her trial in October 2011 she offers Mary as an alibi witness, who testifies that Karen was at work when she is alleged to have committed the crime. On cross examination, the prosecutor seeks to ask Mary "Weren't you convicted in November 2001 of 'filing a false statement in connection with your taxes'"? Assume that the prosecutor has a certified copy of the judgment in Mary's 2001 case, showing she was found guilty on November 15, 2001, for which she received the maximum penalty of eleven months and twenty-nine days incarceration, all of which was suspended. Karen objects that this is inadmissible. How should the judge rule? A. Sustain the objection, because the prior conviction was not punishable by over a year in prison. B. Overrule the objection, because establishing the elements of this crime required proof of an act of dishonesty or a false statement by the witness and is not time barred. C. Overrule the objection, if it readily can be determined that establishing the elements of the crime required proof or admission of an act of dishonesty or false statement by the witness. D. Overrule the objection, since the conviction is not time barred, if the court finds the probative value of admitting the evidence outweighs its prejudicial effect to the accused.

B Answer A is incorrect because convictions that satisfy FRE 609(a)(2) are admissible to impeach, whether or not the sentence that could have been imposed was over one year. The time bar runs from the later of conviction or release, but because the entire sentence was suspended, the release was the same day as the conviction (Nov. 15, 2001). As it is now October 2011, this is at most 9 years, 11 months ago. Answer C is incorrect. While the language from FRE 609(a)(2) does discuss whether it can be readily determined that the elements required proving a dishonest act or false statement, you should be able to recognize that it can readily be determined that establishing the elements of the crime of "filing a false statement" required proof or admission of an act of false statement. Answer D is incorrect. Whether the probative value outweighs the prejudicial effect to the accused is only relevant for a conviction offered under FRE 609(a)(1) - and only those of the defendant, not a witness.

Police respond to a 911 call of a suspicious person in a black coat near an elementary school who approached several children outside the school. Arriving at the school three minutes after the call, the police ask the principal for a description of the suspicious person, and the principal says, "he was short, maybe 5 feet, with a black ¾ length coat and a moustache. He was just here." The police rapidly search but find no one fitting this description. The following week a man exactly fitting this description is arrested near the school while trying to lure a child into a car. The man is charged with attempted child abduction, but by the time of trial the principal has died. The prosecution offers the principal's description from the week before the defendant's arrest as evidence of his plan. The defendant objects that admitting the principal's statement violates the confrontation clause. Which best describes what the judge will likely do? A. Exclude the statement because it was testimonial. B. Admit the statement because its purpose was responding to an ongoing emergency. C. Exclude the statement because it was testimonial and, although the declarant is unavailable, the defendant never had an opportunity to develop the testimony by cross examination. D. Admit the statement because the defendant can cross examine the officer who took the statement.

B Answer B is correct. The admission of this statement does not violate the confrontation clause because, although it has several features that suggest it is testimonial (it was focused on particular suspect, for a specific type of crime), because its primary purpose was to respond to an ongoing emergency of a suspected child abductor in the immediate area, it is not testimonial. Michigan v. Bryant. Although the report was of a "suspicious" person, the suspicion was of an imminent threat to children near the school (given the quick response of the police, the principal saying he was just here, and the rapid search by the police), which show the ongoing nature of the emergency. Answer A is wrong, the statement is not testimonial because its primary purpose was responding to an ongoing emergency. Michigan v. Bryant. Answer C is wrong because the primary purpose in the principal's making the statement was responding to the ongoing emergency of a suspected child abductor in the area, and such statements are not excluded by the confrontation clause under Michigan v. Bryant. Answer D is wrong because the defendant's ability to cross examine someone who took an out of court statement does not satisfy the requirements of the confrontation clause.

John is charged with robbery of a convenience store, and a witness testifies at trial that from across the street she clearly saw him run from the store because it was a clear night with a full moon. John's lawyer asks the witness whether she told a police officer at the scene that she could not see the robber very clearly, which she denies having said. To prove that the witness did not see the robber as clearly as she testified, John's lawyer moves to admit a police report from an interview with the witness at the scene, shortly after the robbery, in which a police officer noted, "Witness says she couldn't see the robber very clearly." The prosecution objects that the police report is inadmissible hearsay. Which best describes the judge's likely ruling? A. Admit the police report as a public record to show that the witness did not see the robber very clearly. B. Exclude the police report as a public record but admit the witness's statement in it for impeachment. C. Admit the police report as a public record because it is a factual finding offered against the government. D. Exclude the police report as a public record because the inconsistent statements by the witness show a lack of trustworthiness in the source of the information.

B Answer choice B is correct. The police report is inadmissible hearsay, but the witness's statement in it is admissible for impeachment. Note that this does not show that the witness "couldn't see the robber very clearly," but only that she has given two different accounts (one to the officer and one at trial) of how well she could see the robber. Answer choice A is wrong. The police report is inadmissible hearsay that does not qualify as a public record because police reports are inadmissible under 803(8)(A)(ii), as this is a "matter observed" by law enforcement personnel. Admitting it to show "that the witness did not see the robber very clearly" is offering it to show the truth of what it asserts. Answer choice C is wrong. The report is not admissible as a public record because police reports are inadmissible under 803(8)(A)(ii), as this is a "matter observed" by law enforcement personnel. It is not a "factual finding" because the report recounts what the witness said, rather than offering any conclusion about it. Answer choice D is wrong. Regardless of the trustworthiness of the record, matters observed by law enforcement while under a legal duty to report are inadmissible in criminal cases.

John is charged with robbery of a convenience store, and a witness testifies at trial that from across the street she could clearly saw him run from the store because it was a clear night with a full moon. To prove that the witness's ability to see the robber would have been reduced because it was a heavily clouded night, with barely a sliver of moon, John's lawyer offers a report from the National Oceanic and Atmospheric Administration, a federal agency charged with monitoring and forecasting weather and astronomical events, showing that at the time, date and location of the robbery it was 100% overcast and the moon was barely a sliver. The prosecution objects that the report is inadmissible hearsay. How should the judge rule? A. Exclude the report because this is a criminal case. B. Admit the report to show the state of the weather and the moon at the time, date, and location of the robbery. C. Exclude the report unless the defendant can show that the source of the information or the circumstances do not indicate a lack of trustworthiness. D. Admit the report but only to contradict the witness and not to show the state of the weather or the moon at the time, date, and location of the robbery.

B Answer choice B is correct. The report is admissible as a public record, as a matter observed by a public office under a legal duty to report, pursuant to Fed.R.Evid. 803(8)(A)(ii). NOAA is a public office ("a federal agency") under a legal duty to report ("charged with monitoring and forecasting weather and astronomical events"). These are admissible in a criminal case because NOAA is not "law-enforcement personnel". Therefore, answer choice A is wrong. The report is admissible to show the actual state of the weather and the moon at the time and place of the robbery. Note that introducing this to "contradict" the witness requires it be admitted to show its truth. It doesn't contradict the witness if it is not admitted for its truth. Therefore, answer choice D is wrong. The burden to show a lack of trustworthiness from the circumstances or source of the information is on the opponent of the record. The proponent need not rebut this unless it is raised. See Advis. Comm. Notes to 2014 Amendment to 803(8)(B). Therefore answer choice C is wrong.

John is charged with robbery of a convenience store, and the clerk testifies that just before the robbery, John bought a pack of cigarettes that the clerk rang up on the register. To prove the time of the robbery, the prosecution offers the cash register tape, showing a sale of one package of cigarettes for $7.00, at 11:47 pm on November 10, 2018. The clerk testifies that he changes the tape when it turns pink, indicating it will soon run out, and that he stores the tapes in the safe where the accountant picks them up every month. The clerk testifies that he enters sales when they are made, as this is the only way to open the cash register drawer, and that he doesn't make sales without entering them on the register. He admits that he doesn't know the process by which the cash register records the prices for items sold and cash received when the keys are pressed, but he testifies that the accountant has never questioned the accuracy of the accounts when reconciling the cash receipts. John objects that the register tape is hearsay. Which best describes the judge's likely ruling? A. Exclude the register tape because the clerk does not know how the register records prices for items sold and cash received when the keys are pressed. B. Admit the register tape because it is a record of a regularly conducted activity. C. Exclude the register tape unless the accountant can also testify to not questioning the accuracy of the accounts when reconciling the cash receipts. D. Admit the register tape because it is a recorded recollection of the clerk making the sale.

B Answer choice B is the correct answer. The register tape is admissible as a record of a regularly conducted activity of a business, namely the cash register sales of the store, under 803(6). It is made at or near the time of the event (the same of the cigarettes) because the clerk testifies that every sale must be rung up in order to open the cash drawer. Ringing up sales is a regularly conducted activity, because the clerk testified that no sales are made without ringing them up, and this also shows it was the regular practice of the clerk to make these records. Answer choice A is wrong because the clerk need not know how the cash register works to establish the foundation, unless the opponent can show some reason that the method lacks trustworthiness. This is rebutted by the accountant's not finding any problem with the accounts and the cash receipts. Therefore, answer choice C is wrong. Answer choice D is wrong because the clerk did not testify that he could not remember, so this evidence is not being admitted as a past recorded recollection.

Mary is charged with embezzlement from her employer by pocketing money from the cash drawer, and the prosecution calls a co-worker, Sharon, to testify she often saw Mary "rearranging" the cash drawer for no apparent reason. Mary seeks to ask Sharon on cross examination, "Didn't you often steal office supplies from your workplace?" The prosecution objects. How should the judge rule? A. This is inadmissible character evidence as impeachment because it is in an improper form. B. This is admissible character evidence as impeachment to show Sharon is dishonest. C. This is inadmissible character evidence because it involves an uncharged crime. D. This is inadmissible character evidence because Sharon is not the defendant.

B Correct answer is B. This is proper impeachment by specific acts showing a character for untruthfulness on cross examination. Anytime a witness testifies, his or her character for truthfulness is thereby in question, so impeachment by showing a character for untruthfulness is proper. Fed.R.Evid. 608(a).Links to an external site. However, this impeachment must be by reputation or opinion evidence only. Proof of specific acts of untruthfulness is not permitted. Fed.R.Evid. 608(b)Links to an external site.. However, the witness (or a character witness who testified about the witness's character) may be asked about specific acts on cross examination only, in order to test the witness's character for truthfulness (or the witness's knowledge of the character of another witness about whose character the witness has testified). A. is wrong because this impeachment evidence through showing character for dishonesty is in the proper form. Specific acts may be asked of the witness on cross examination. Fed.R.Evid. 608(b). C. is wrong because the specific acts need not be crimes (nor need they be convictions, or even charged crimes). They can be anything that is probative of a character for truthfulness or untruthfulness (such as stealing office supplies). D. is wrong because impeachment by character evidence of a witness is proper in any type of case under Fed.R.Evid. 608(b).

During his lunch hour, Wally witnesses a horrific car crash in which several people were seriously injured. Because Wally was so shaken up from observing the accident, he left work early and took the hour-long train ride home. When he got home his wife Jen asked, "How was your day honey?" Wally responded, "I saw a woman and two young children almost killed by a man who ran a red light. He just plowed through the intersection." At a later trial the victims of the car accident sue the driver for negligence. They want to call Wally to testify. Wally says he would prefer not to testify because the incident was so upsetting to him. Plaintiffs call Jen to testify as to what Wally told her about the accident. As the defense attorney, what is your BEST argument for keeping Jen's testimony out? A. Jen cannot testify because she lacks personal knowledge of the accident. B. Wally's statement was not made while he was still under the stress of the excitement. C. Because Wally was not the actual victim of the car accident, it cannot be a stressful event for him. D. Wally cannot testify as to legal conclusions of fault.

B The best objection is B. The facts to not indicate that Wally is still under stress of the event at the time he told Jen what he saw. All of the other objections are not valid objections to statements admitted as excited utterances under 803(2). Answer choice A is not a good objection because while the declarant (Wally) must have personal knowledge of the facts, the witness (Jen) who testifies about what Wally said, does not have to have independent personal knowledge. Answer choice C is not a good objection. Clearly Wally could be upset about what he observed even though he was not involved in the accident. Answer choice D is not a strong objection because Wally is not making legal conclusions of fault, but he is just testifying as to what he observed.

Plaintiff sues Defendant for injuries Plaintiff suffered when he slipped and fell in Defendant's store. In order to prove the injuries Plaintiff calls her ex-husband to testify about the back pain Plaintiff experienced immediately after the accident. Ex-husband testifies that Plaintiff acted just fine after the accident and never complained about any pain or soreness to her back. Plaintiff now wants to offer the deposition testimony of ex-husband given in this case in which ex-husband stated that Plaintiff was bedridden for several weeks after the accident and was in constant pain. This deposition testimony is: A. Inadmissible hearsay. B. Admissible both to impeach the ex-husband's in-court testimony and to prove that Plaintiff was bedridden and in pain after the accident. C. Admissible only to impeach the ex-husband's in-court testimony but not to prove the truth of the matter asserted in the deposition. D. Inadmissible unless the ex-husband refused to testify and was deemed unavailable by the court.

B The correct answer choice is B. Rule 801(d)(1)(A) states that if the declarant is now testifying and subject to cross-examination about the prior statement and the prior inconsistent statement was made at a deposition, that prior statement is not hearsay. Answer choice A is wrong because Rule 801(d)(1)(A) defines this statement as not hearsay. Answer choice C is wrong because the deposition testimony can be used for more than just impeaching the witness. It can be used to prove the truth of what the statement asserts. Answer choice D is wrong. The deposition testimony is admissible under these facts because the ex-husband is testifying in this case.

Defendants Robert and Renee are co-owners of the Greasy Spoon, a local restaurant. They are prosecuted for racketeering and mail fraud. At trial, the prosecution offers a statement made by Renee that was overheard by a restaurant diner. The diner would testify that about three months before their arrest he overheard Renee say to Robert "we really need some Swiss bank accounts to handle these questionable wire transactions." Renee's lawyer objects this is hearsay. Assuming no other facts have been proven, which is the prosecution's strongest basis to admit this testimony? A.The testimony is admissible as a statement against interest. B. The testimony is admissible as a statement by a party opponent. C. The testimony is admissible as a statement by a co-conspirator. D. The testimony is admissible because it is not offered to prove the truth of the matter asserted.

B The correct answer is B. Renee made the statement and is being used against her. Statements by a party opponent come in with no other limitations, and no additional foundational requirements need be established.Answer choice A is not correct as the prosecution must also establish that the declarant is unavailable, that it so far exposed a declarant to criminal liability that a reasonable person in the declarant's position would not have made it were it not true and that corroborating circumstances clearly indicate its trustworthiness. 804(b)(3)(A) and (B). Answer choice C is not the best theory of admissibility because this exception too has additional foundational requirements that 801(d)(2)(A) does not have, such as proving that a conspiracy existed and that the statement was made in furtherance of the conspiracy, while the conspiracy was on-going. In addition, because this is Renee's own statement it is not the testimony of Renee's co-conspirator.Answer choice D is not a good theory of admissibility because the prosecution would want to use this statement to prove the truth of the matters asserted in the statement and FRE 801(d)(2)(A) allows them to admit the statement for its truth.

Which best describes Sam's testimony, if Karen raises a hearsay objection? To prove that Karen was guilty of forgery, the government offers a witness named Sam, who testifies, "Karen's boss told me six months ago that 'Karen is always practicing writing and signing stuff so that it looked like other folks' handwriting.'" A. Sam's testimony is not hearsay, because it is not an assertion but merely recounts observations. B. Sam's testimony is inadmissible hearsay. C. Sam's testimony is hearsay, but it is admissible under the functional exception as a verbal act. D. Sam's testimony is hearsay, but it is admissible as an admission of the agent, servant or employee of a party opponent.

B The correct answer is B. The declarant here is Karen's boss, and the statement is "Karen is always practicing writing and signing stuff so that it looked like other folks' handwriting." If offered to prove that she is doing this (i.e., practicing forgery), it is hearsay. Because it is an assertion, answer choice A is incorrect. Further, Karen's actions are not hearsay because her practicing signatures is not an assertion, so it is not a statement. Answer C is incorrect. This is not a verbal act, as the boss's statement does not have independent legal significance. Answer D is incorrect. Karen's boss is not an agent, servant or employee of the party opponent, thus FRE 801(d)(2)(d) is inapplicable - the boss is the employer of the party opponent. Note that if the boss were the witness, there would be no hearsay problem with the boss describing the defendant's actions.

In a products liability suit for a plaintiff smoker's cancer, Defendant tobacco company offers the government-mandated label from its cigarette package, "Smoking May Cause Cancer," to rebut the plaintiff's claim that he was misled about the health risks of smoking. If the plaintiff smoker objects that this is hearsay, which best describes how the judge should rule? A. Overrule the objection, because although the label is hearsay, it is a statement made by a party in the case, therefore it is admissible as a definitional exception. B. Overrule the objection, because although the label is hearsay, it is admissible for its effect on the listener. C. Overrule the objection, because although the label is hearsay, it is admissible as a verbal object to identify the cigarette at issue. D. Sustain the objection, because the label is an out-of-court statement offered for the truth of the matter asserted, and is therefore inadmissible hearsay.

B The correct answer is B. The label is an out-of-court statement, but it is not offered for the truth of what it asserts (i.e., that smoking may cause cancer), so answer D is incorrect. Instead, it is being offered to show that the plaintiff was warned of the risk, so it is offered to show its effect on the listener (or here, the reader of the warning). Answer A is incorrect. This is not admissible as a statement of an opposing party, under 801(d)(2)(A), because here the declarant (defendant) is offering its own statement (i.e., the label on its own product). There is also an argument that it's not even a statement of a party, because as "government-mandated" it is arguably a statement by a government agency. Answer C is incorrect. The label could be offered as a verbal object, IF identification of the cigarette was at issue - but the question gives you a different reason that the label is offered.

Consider the following evidence: The words "In God We Trust," on a dime, offered by a taxpayer challenging the coin's design as a violation of the clause in the constitution's 1st Amendment that "Congress shall make no law respecting an establishment of religion." Assume that this is understood to mean either creating a government religion or government endorsement of religion. If the government objects that this is inadmissible hearsay, which best describes how the judge should rule? A. Sustain the objection, because the words are an out-of-court statement offered for the truth of the matter asserted, and therefore inadmissible hearsay. B. Overrule the objection, because the words are not offered for the truth of the matter asserted. C. Overrule the objection, because the words are admissible as a verbal object, to identify the coin. D. Overrule the objection, because although the words are hearsay, they are admissible for their effect on the listener.

B The correct answer is B. This is an out of court statement that is not offered for the truth of the matter it asserts. A is incorrect because the words on the coin are offered to show that the government has made a law respecting an "establishment of religion," not that, in fact, anyone actually does "trust" in god. (FRE 801(c)). C is incorrect. Identification of the coin (i.e., the statement's use as a verbal object) is not relevant in this case because the challenge is to the design of the coin rather than to anything concerning any particular dime. This would be correct if identification of the coin (e.g., as the coin somehow harmed a plaintiff by injuring him, etc.) were the reason for the statement's admission - but it's not. There is no legal significance of the effect on the person who reads the words on the coin, so D is incorrect.

Plaintiff sues Defendant in a slip and fall case. To prove that Defendant did not put up warning signs around the slick area of the floor, Plaintiff calls Witness. Witness will testify that when she walked into the store and saw Plaintiff lying on her back crying and shaking, Plaintiff said, "Wow that floor is slick and I did not see any warning signs." Defense objects that the statement is inadmissible hearsay. How should the judge rule? A. Sustain the objection because the statement is inadmissible hearsay. B. Overrule the objection because the statement is an excited utterance. C. Sustain the objection because the statement does more than describe the event. D. Overrule the objection because the statement is not being offered for a hearsay purpose.

B The correct answer is B. This statement fits the elements of an excited utterance under Rule 803(2) because a stressful event happened, and the declarant was still under the stress of the event when she made her statement relating to the startling event. Answer choice A is wrong because an exception does apply. Answer choice C is wrong because the excited utterance exception allows statements related to the event and all of the declarant's statements are related to the event. This is broader than the present sense impression exception where the statement must describe the event. Answer choice D is wrong because the facts state that the statement is being offered for a hearsay purpose.

Plaintiff slips in Defendant's store, and sues Defendant claiming Defendant failed to maintain a safe floor for shoppers. A doctor, who treats Plaintiff, testifies as follows: "Plaintiff consulted me regarding episodes of back pain. He said they started when he fell down in the store. Apparently, there was a big puddle of soapy water that made the floor slick." Defendant objects that the physician's testimony is inadmissible hearsay. Which best describes the judge's likely ruling? A. Admit the entire statement of the plaintiff because it is a statement made for medical diagnosis or treatment. B. Admit the statement concerning how the pain began because it is a statement made for medical diagnosis or treatment but exclude the plaintiff's description of the puddle. C. Admit the entire statement of the plaintiff unless the doctor is a consulting physician. D. Admit the entire statement of the plaintiff because it is a statement of the declarant's state of mind.

B Answer choice B is correct. The plaintiff's statements to the doctor are statements for medical diagnosis or treatment (803(4)), including the inception of the cause "insofar as reasonably pertinent to diagnosis or treatment," but the puddle of soapy water detail is probably not reasonably pertinent to diagnosis or treatment, so that will be excluded. Therefore answer choice C is wrong. Answer choice A is wrong. Fed.R.Evid. 803(4) ACN's distinguish between admissibility of how the injury arose (being struck by a car) but not further details (that the car was driven through a red light). While the entire statement might satisfy 803(4)(B) as describing the inception or general cause of the symptoms, and it is made for diagnosis or treatment, the detail about the puddle is not reasonably pertinent to diagnosis or treatment. Answer C is wrong. 803(4) applies equally to consulting physicians. Answer choice D is wrong. The argument that plaintiff's pain is a statement of his state of mind might be a plausible answer if the plaintiff described "then existing" back pain, under 803(3). Because the plaintiff is describing "episodes he has had of back pain," it is not as clear that he is describing then-existing bodily sensation. Moreover, the "entire statement" clearly encompasses more than state of mind, since it includes the description of the puddle.

Bill is charged with Operating a Motor Vehicle Under the Influence of Alcohol (OUI), 4th offense, an element of which is that the defendant was convicted of OUI three times before the date of the alleged offense. What is the BEST objection Bill can make against the admission of the three prior OUI convictions? A. Proof of the three prior OUI's is cumulative, because the prosecution will already be able to prove the facts of Bill's guilt in this most recent OUI. B. Proof of the three prior OUI's poses a high risk of unfair prejudice, because the jury may think Bill has a propensity to drive under the influence, and they will convict him in the present case based on this. C. Proof of the three prior OUI's poses a high risk of confusion, because the jury may think they are evidence of Bill's guilt in this case. D. Proof of the three prior OUI's has no probative value to this case so it is not relevant

B.

Defendant is on trial for aggravated assault. To prove that Defendant is the perpetrator of the crime, Prosecution wants to introduce Defendant's three prior criminal convictions. One prior conviction is for misdemeanor shoplifting and two of the prior convictions are for felony assault. Defense objects to the admission of all prior convictions. How should the judge rule? A. Admit all three convictions because they all show a pattern of criminal misconduct. B. Admit the two assault convictions because they are similar to the crime at issue in this case but exclude the shoplifting conviction because it is irrelevant. C. Exclude all of the convictions because the evidence could cause the fact finder to decide the case on impermissible grounds. D. Exclude all of the convictions because they are not relevant to any issues in this case.

C

Defendant is on trial for his involvement in a bank robbery. During the investigation of the case, Defendant was questioned by the police after waiving his right to remain silent. During that interrogation the police told Defendant that they would make a favorable recommendation to the prosecutor if Defendant "came clean" and told them about his involvement in the robbery. Defendant told the police, "I would like to help you. I was only a small fish. But if I tell you anything the others will make real trouble for me." At trial the prosecution wants to offer Defendant's pre-trial statement to help establish his involvement in the robbery. Defense objects. How should the judge rule? A. Overrule the objection and allow the statements to impeach the Defendant if he testifies and claims that he was not involved in the robbery. B. Sustain the objection because Defendant's statements were part of plea discussions. C. Overrule the objection because Defendant's statements were not part of plea discussions. D. Sustain the objection because the statements cannot be corroborated by other evidence in the case.

C

In a suit under the Fair Housing Act, which forbids discrimination based on race, sex, religion, national origin, disability or the presence of children in sale or rental of housing, plaintiff buyer claims he made a higher offer for a property than thirty other buyers, yet seller rejected his offer in favor of a lower offer by a buyer of a different race. Plaintiff seeks to introduce thirty charts, each of which shows his offer versus another buyer's offer in the form "plaintiff's offer amount > another offer amount," indicating that the plaintiff's offer was higher than the alternative. Defendant, conceding plaintiff's offer was higher than all other offers, objects under Rule 403. Which best describes how the judge should rule? A. The evidence is not relevant, because other factors beside the price legitimately affect a seller's decision to accept an offer. B. The evidence is relevant but poses a high risk of misleading the jury that the seller must accept the highest offer which substantially outweighs its probative value. C. The evidence is relevant but poses a high risk of wasting time given the method of presenting the information that substantially outweighs its probative value. D. The evidence is not relevant, because the defendant has conceded the plaintiff made the highest offer.

C

Jane sues American Motors, which manufactured her car, claiming breach of the implied warranty of merchantability (that the car is suitable for its intended use) because the car's engine failed after two years and 20,000 miles. American Motors defends that the engine failed because Jane did not follow its recommendation to change the oil every six months or 5,000 miles. Jane offers a receipt, from a year after she bought the car, for an oil change and some additional work on her car performed at a service station. American Motors objects that this evidence is not relevant. With respect only to the issue of relevance, how should the judge rule? A. The receipt is relevant because it can show another cause for the engine's failure related to the additional work performed on the car. B. The receipt is not relevant without evidence that Jane also changed the oil prior to and after the instance memorialized by the receipt. C. The receipt is relevant as evidence that Jane changed the car's oil. D. The receipt is not relevant without evidence as to what the additional work performed on the car was.

C

Karen is charged with a crime that occurred March 1, 2001, and calls Sam as an alibi witness in her February 2011 trial. Sam testifies that Karen was at work with him on March 1, 2001. On cross examination, the prosecutor seeks to ask, "Sam, weren't you convicted in April 2002 of burglary?" Assume that the prosecutor has a certified copy of the judgment in Sam's 2002 burglary case, showing he was found guilty, taken into custody, and sentenced to five years in prison on June 3, 2002, which sentence he served in full. Karen objects that this is inadmissible. How should the judge rule? A. Overrule the objection, because establishing the elements of burglary required proof or admission of an act of dishonesty. B. Overrule the objection, because burglary is a felony. C. Overrule the objection, because the conviction occurred less than ten years before and the punishment for burglary was sufficiently severe and its admission is not outweighed by the risk of unfair prejudice or confusion of the issues. D. Sustain the objection, because although the conviction had a sufficiently severe punishment the conviction is too old.

C Answer A is wrong. The conviction does have a sufficiently severe punishment to satisfy FRE 609(a)(1)(A) (since Sam was sentenced to five years in prison, it clearly carries over a year's imprisonment.). The prior conviction is not, however, admissible under FRE 609(a)(2), as the elements of burglary do not require stablishing proof of an act of dishonesty or a false statement. While burglary is a felony, Answer B is not the best answer because FRE 609(a)(1) does not require a crime be a "felony" but that it be punishable by over a year in prison or death. Also, this is not the most complete answer, because it does not address the time bar issue. Answer D is incorrect. The conviction also satisfies the time bar in FRE 609(b). Note that FRE 609(b) measures the ten year period from the date of conviction or release of the witness from confinement, whichever is later. Since Sam was released from prison June 3, 2007 (note he was "found guilty, taken into custody, and sentenced to five years in prison on June 3, 2002, which sentence he served in full."), and the trial is in February 2011, this conviction is only four years old.

Police investigate a bar fight between two friends, Cheryl and Mary. They interview Cheryl, who says "Mary hit me in the back of the head with a bottle." The police charge Mary with assault, and over the next several months Mary tries to convince Cheryl that the fight was a big misunderstanding that they should put behind them and settle without the courts or the police. Cheryl declines to do this, so the week before trial, as a "peace offering," Mary gives Cheryl an all-expenses paid trip for a six month round the world cruise that leaves the day before the trial is scheduled to start. At trial, Cheryl is away on the cruise, so the prosecutor offers a police officer to testify to the statement Cheryl made to the police that "Mary hit me in the back of the head with a bottle." Mary objects that admitting this statement violates the confrontation clause. Which best describes what the judge will likely do? A. Admit the statement because it is supported by sufficient guarantees of trustworthiness, considering the totality of circumstances under which it was made, and it is more probative than any other evidence the prosecution can obtain through reasonable efforts. B. Exclude the statement because the police were not responding to an ongoing emergency when the statement was made. C. Admit the statement because Mary's actions made Cheryl unavailable and were done with that intent. D. Exclude the statement because it is testimonial, Cheryl is unavailable, and Mary never had a chance to cross-examine Cheryl.

C Answer C is correct. The statement will be admitted because Mary procured Cheryl's absence by wrongdoing, and by wrongfully making Cheryl unavailable Mary forfeits the protection of the confrontation clause. Giles v. Calif. 554 U.S. 353 (2008). Mary's action of giving Cheryl a ticket for a six-month trip departing the day before the trial is set to begin clearly shows an intent to make her unavailable. This is heightened by her effort to persuade Cheryl for several months that the fight was a big misunderstanding.

Police investigate a bar fight between two friends, Cheryl and Mary. They interview Cheryl, who says "Mary hit me in the back of the head with a bottle." The police charge Mary with assault, and over the next several months Mary tries to convince Cheryl that the fight was a big misunderstanding that they should put behind them and settle without the courts or the police. Cheryl declines to do this, so the week before trial, as a "peace offering," Mary gives Cheryl an all-expenses paid trip for a six month round the world cruise that leaves the day before the trial is scheduled to start. At trial, Cheryl is away on the cruise, so the prosecutor offers a police officer to testify to the statement Cheryl made to the police that "Mary hit me in the back of the head with a bottle." Mary objects that this statement is inadmissible hearsay. Which best describes what the judge will likely do? A. Admit the statement because it is supported by sufficient guarantees of trustworthiness, considering the totality of circumstances under which it was made, and it is more probative than any other evidence the prosecution can obtain through reasonable efforts. B. Exclude the statement because Cheryl is not unavailable since the prosecution did not attempt to depose her or procure her testimony through process or other reasonable means. C. Admit the statement because Mary's actions made Cheryl unavailable and were done with that intent. D. Exclude the statement because it is testimonial, Cheryl is unavailable, and Mary never had a chance to cross-examine Cheryl.

C Answer C is correct. The statement will be admitted because Mary procured Cheryl's absence by wrongdoing, and by wrongfully making Cheryl unavailable Mary forfeits the protection of the rule against hearsay. 804(b)(6).Mary's action of giving Cheryl a ticket for a six-month trip departing the day before the trial is set to begin clearly shows an intent to make her unavailable. This is heightened by her effort to persuade Cheryl for several months that the fight was a big misunderstanding.

John is charged with robbery of a convenience store, and the clerk testifies that just before the robbery, John bought a pack of cigarettes that the clerk rang up on the register. As the clerk turned back with the change, John pointed a gun at him and say, "Give me the money and you won't get hurt!" After the clerk emptied the cash drawer into a bag and gave it to John, John said "get on the floor, and don't move." After the clerk heard John leave, he dialed 911 and reported the robbery. On the back of a candy wrapper he wrote, "Give me money," "5 feet 10 or 6 feet," "red hat," "silver gun with black handle". When the police arrived three minutes later, he gave a description of John and the wrapper with his notes, saying "I've worked here ten years and that's never happened." On arrest, John is wearing a red hat, has a silver gun with a black handle and is 5 feet 11 inches tall. At trial a year later, the clerk cannot recall whether John was wearing a hat during the robbery or the color of the gun. The prosecutor asks if his memory is exhausted about these details. The clerk testifies that it is, but he thinks these details were on the notes he made at the time on a candy wrapper which he gave the police. When the prosecutor shows the clerk the candy wrapper, the clerk identifies it as the one he gave the police. The prosecutor moves to admit the candy wrapper with the notes into evidence to show the robber wore a hat and the color of the gun. John objects that the notes on the candy wrapper are hearsay. The prosecutor argues the notes are admissible as a recorded recollection. How should the judge rule? A. Admit the notes as a recorded recollection but they may only be used to corroborate the clerk's testimony. B. Exclude the notes as a recorded recollection because the clerk is available to testify. C. Allow the notes to be read to the jury but admit them as a recorded recollection only if John offers them. D. Exclude the notes as a recorded recollection because the clerk never adopted them.

C Answer choice C is correct. The notes may be read to the jury as a recorded recollection under 803(5), but the notes may be admitted into evidence only if John offers them. The notes concerned a matter about which the witness once knew about but now cannot recall well enough to testify fully and accurately. The notes were made by the witness when it was fresh in the witness's memory. The notes accurately reflect the witness's knowledge, as shown by the congruence between John's height, hat and gun color and these details on the witness's notes. The witness cannot now recall well enough to testify fully and accurately. Answer choice A is wrong because recorded recollections may be admissible as substantive evidence on their own rather than merely to corroborate. As they may only be admitted when the witnesses cannot recall fully and accurately, they substitute for - rather than corroborate - the witness's testimony. Answer choice B is wrong for that same reason. Answer choice D is wrong because the rule only requires the witness to verify that at the time the record was made the information was fresh in the witness's mind and the witness then accurately recorded the information.

Police respond to a 911 call of a suspicious person in a black coat near an elementary school. Arriving in three minutes at the school, they ask the principal for a description of the suspicious person, and she says, "he was short, maybe 5 feet, with a black 3⁄4 length coat and a moustache. He was just here a moment ago." They find no one in the area fitting this description, but the following week a man exactly fitting this description is arrested near the school trying to lure a child into a car. The man is charged with attempted child abduction, but by the time of trial the principal has died. The prosecution offers the principal's description from the week before the defendant's arrest as evidence of his plan. The defendant objects that the principal's statement is inadmissible hearsay. Which best describes what the judge will likely do? A. Admit the statement, as it is not offered for the truth of the matter but to show the defendant's plan. B. Admit the statement, as it is a statement of an intent to engage in future conduct. C. Admit the statement, as it is a present sense impression. D. Admit the statement, as it is a recorded recollection.

C Answer choice C is correct. The statement is admissible as a present sense impression. Fed. R. Evid. 803(1). The statement is made shortly after the event (the police come in three minutes and the principal says he was "just here." She is describing the event. Answer choice is wrong. The statement is offered for the truth of the description of the person near the school. Answer choice B is wrong. The statement is not a statement of intent to engage in future conduct; it is a description of someone. The prosecution wants to admit it to show the defendant's plan, but the description itself shows nothing about a plan. Answer choice D is wrong because the declarant is not claiming they don't remember a past recollection. In fact, the declarant is not even in court testifying.

Carl is charged with robbery and claims he has been misidentified. At a preliminary hearing the victim, Marsha, testifies how the robber held her at knifepoint and took her wallet. Because Carl does not want to be seen by Marsh when she describes her robber, Carl's lawyer asks that Carl be kept out of the courtroom during Marsha' testimony, which the judge agrees to. Marsha describes the robbery and the robber in minute detail, including a small dragon tattoo on the robber's right hand, his slight limp, and a scar on his left cheek. These features match Carl precisely. After Marsha's testimony, Carl's lawyer decides that any further questioning will simply provide a more detailed description and declines to do any cross examination. By Carl's trial, unfortunately Marsha has died. The prosecutor offers her testimony from the preliminary hearing and Carl objects that this violates the confrontation clause. How should the judge rule? A. Admit the testimony because it is a statement of one identifying a person after perceiving them. B. Exclude the testimony because Carl's lawyer did not cross examine Marsha at the preliminary hearing and she is now unavailable. C. Admit the testimony because Carl's lawyer had an opportunity to cross examine Marsha at the preliminary hearing. D. Exclude the testimony because Carl's lawyer did not have a similar motive to cross examine Marsha at the preliminary hearing.

C C is correct. The confrontation clause requires an opportunity for cross examination, which Carl had at the preliminary hearing. Despite Marsha's unavailability, the prior testimony is admissible because Carl had the chance to cross examine her. Calif v. Green, Crawford v. Washington.

Mary is charged with embezzlement from her employer by pocketing money from the cash drawer, and the prosecution calls a co-worker, Sharon, to testify she often saw Mary "rearranging" the cash drawer for no apparent reason. Mary's lawyer asks one question on cross-examination of Sharon: "Are you sure it was Mary you saw rearranging the cash drawer and not someone else?" The prosecution then seeks to call Marsha, a coworker of Sharon's, to testify "Sharon is the most honest person I know." Mary objects. How should the judge rule? A. This is inadmissible character evidence because it is in an improper form. B. This is inadmissible character evidence because the witness is only offered to increase Sharon's credibility. C. This is inadmissible character evidence because the witness's character for truthfulness has not been attacked. D. This is admissible character evidence because it is in a proper form and this is a case of embezzlement.

C Correct answer is C. This is inadmissible character evidence because the witness's character for truthfulness has not been attacked. When a witness testifies, their honesty is thereby at issue. If their character for truthfulness is attacked, either by vigorous cross examination suggesting that they have lied or through reputation or opinion evidence that they have a character for untruthfulness, then the party that called the witness may respond with character evidence showing their character for truthfulness. Fed. R. Evid. 608(a). Links to an external site. This can be in the form of reputation or opinion testimony (here it is in the form of opinion). But Sharon's character for truthfulness has not been attacked. Note that the cross examination suggested she was mistaken - not that she was lying. Because Sharon's character for truthfulness has not been attacked by Mary's lawyer, the prosecution may not introduce a character witness to show Sharon's character for truthfulness. A. is wrong because this character evidence is in the proper form (opinion), but it is not admissible because the witness's (Sharon's) character for honesty has not yet been attacked. B. is wrong because a character witness, if character evidence is proper, may be offered to support a witness's credibility by showing a character for honesty. The witness's character for honesty must be first be attacked before this is proper. D. is wrong because although the character evidence is in a proper form, the nature of the charge is irrelevant (and, of course, Sharon's character for honesty has not yet been attacked).

Middle-aged Plaintiff Paul sues his high school coach, alleging coach sexually abused him as a child. In support of this claim, Paul offers testimony from his best friend, William, to whom he disclosed this abuse. William will testify that Paul told him in great detail about his memories of having been abused, and can recount precisely what Paul said. Defendant coach objects that this is hearsay. Assuming Paul does not testify, how should the judge rule? A. Exclude William's testimony because, as Paul's best friend, he has such bias that he cannot be credible. B. Exclude William's testimony because it is both hearsay and not relevant. C. Exclude William's testimony because it is inadmissible hearsay. D. Admit William's testimony because it is circumstantial evidence of Paul's memory that the abuse occurred.

C The correct Answer is C. William's testimony, offered to prove that the allegations are true, is hearsay. Answer A is incorrect. While bias reduces the weight of a witness's testimony, it is not a basis to exclude it. Answer B is incorrect, as William's testimony is relevant: it makes it somewhat more likely that what plaintiff alleges actually happened. Answer D is incorrect. This is not circumstantial evidence of memory - this is direct evidence that the memories are accurate - which is hearsay. Witness's testimony is hearsay if offered as substantive evidence that the abuse occurred.

Plaintiff sues Defendant for injuries sustained in a car accident. To prove the extent of the injuries Plaintiff offers the emergency room report that was prepared when Plaintiff was brought into the hospital immediately after the accident. The report states: Patient appears to be in shock. There are numerous lacerations on arms and legs. Friend accompanying Patient reports that injuries were the result of a car accident. A Records Custodian from the hospital will testify that these emergency room reports are prepared as part of the hospital's regular practice, and that physical observations of the patient are completed by the attending emergency room physician. The reports are done soon after the patient is admitted, and the reports are maintained in the hospital's files. This report is: A. Inadmissible hearsay unless the attending physician will testify at trial. B. Admissible in its entirety as a business record. C. The portion of the report that includes the statement of the friend should be redacted because it is not a business record. D. The portion of the report that includes the statement of the friend should be redacted because it is a legal conclusion.

C The correct answer is C. The portion of the statement that deals with medical symptoms prepared by the attending physician soon after the observations were made meets the requirements for the business record exception under Rule 803(6). Answer choice A is wrong because the person who prepared the report does not have to testify as long as someone with knowledge of how the reports are prepared testifies. Under these facts, the Records Custodian has sufficient knowledge. Answer choice B is wrong because the portion of the report made by the friend does not fit the business record exception because the friend is not an employee of the hospital and is not within the scope of the business. Answer choice D is wrong because it is not a legal conclusion. It is not attributing fault. The friend's statement should be redacted from the business record, but for other reasons.

Bob, Carol, Ted and Alice are charged with importing illegal drugs from overseas. Allegedly, Bob organized shipments, Carol went abroad and brought back the drugs, Ted sold them, and Alice kept the group's records. During one trip abroad, Carol is waiting at an airport to fly overseas while her plane is delayed. At the airport bar, the bartender asks if she wants another drink. Carol replies, "Well, since I'm about to take a four-hour flight, come back tomorrow, and it will make me $100,000, I guess I can afford another drink. Sure." All four are prosecuted for drug trafficking, and at their trial the prosecution offers the bartender to testify to Carol's statement. All four object that the testimony is inadmissible hearsay. How should the judge rule? A. Admit the statement against all four defendants as the statement of a coconspirator. B. Admit the statement against Carol but exclude the statement against Bob, Ted and Alice unless there is evidence that establishes the existence of a conspiracy. C. Admit the statement against Carol but exclude the statement against Bob, Ted and Alice. D. Exclude the statement against all four defendants as inadmissible hearsay.

C The correct answer is C. The statement is admissible against Carol, but it is inadmissible against the others as a statement of a coconspirator because it was not made in furtherance of the conspiracy. Answer A is incorrect because even if there is evidence to establish a conspiracy by a preponderance, the statement does not satisfy the requirement that it furthers the conspiracy. It is of course a statement of a party opponent when offered against Carol. Answer B is wrong for the same reason as A. Answer D is wrong because the statement is admissible against Carol as the statement of a party opponent

Which best describes this evidence, if the Defendant raises a hearsay objection? In Defendant's burglary trial, the government offers a police officer who testifies that the serial number on the back of a TV found in defendant's car read "1234567," when Defendant is prosecuted for burglary of victim's home in which a TV with serial number "1234567" was stolen. A. This is hearsay, but it is admissible as an admission of a party opponent, since the TV was found in the defendant's car. B. This is hearsay, but it is admissible as circumstantial evidence of the officer's memory of the TV's serial number. C. This is not being offered for a hearsay purpose and it is admissible as a verbal object identifying the TV as one taken in the burglary. D. This is not hearsay, because it recounts numbers rather than words.

C The correct answer is C. The statement is admissible as a verbal object. The officer's testimony recounts the out-of-court statement "1234567," which is an assertion that this TV had that serial number - which was the same serial number as the victim's TV (imagine if the label on the TV read, instead of a serial number, "This is the Property of the Victim."). It is admissible not to prove that this was the serial number, but to identify it as the same TV taken from the Victim's home. This is using the statement of the numbers as a verbal object, to identify this TV. (Note that the government would need to offer testimony by the victim (or some record, i.e., a writing) of the stolen TV's serial number.). (Note the admissibility would be much more difficult if the label had read "Property of the Victim," because there would be a significant risk under 403 that the jury would use this out of court statement not to identify the TV as one belonging to the victim but as evidence the Defendant stole it). This is not a statement used as circumstantial evidence of the officer's memory, so answer B is incorrect. If it were, then every hearsay statement would be admissible on this ground (i.e., a witness could testify "I remember saying..." as circumstantial evidence of the witness's memory). The officer's memory of the serial number has no relevance here. This is not admissible as a statement of a party opponent, because it is not the defendant's statement, so answer A is incorrect.

Which best describes this evidence if the opposing party raises a hearsay objection? In order to prove that the "Quickie-Mart" was open at 9:00 a.m., testimony by a witness that he saw several customers enter the store at 9:00 a.m. A. This is hearsay, but admissible as a functional exception because entering the store is a verbal act. B. This is hearsay, but admissible as a functional exception because entering the store is a verbal object. C. This does not meet the definition of hearsay, thus the hearsay rule does not preclude its admissibility. D. This is hearsay, but it is admissible under the definitional exception as a statement of identification of a person by a testifying declarant after perceiving the person.

C The correct answer is C. This is non-assertive, non-verbal, conduct; the customers did not go in to show that the store was open, thus it does not qualify as a "statement." This is not a verbal act because the entry to the store has no independent legal significance. This is not a verbal object because the entry to the store is not a statement that is identifying something; the witness is recounting an action. This is not a statement of identification because the witness is describing what he saw, not what he said, and he does not identify the persons but rather describes what the persons did.

Defendant Danny is charged with possession of a controlled substance with intent to distribute. Willie Weasel was also charged for this crime as a co-defendant. Weasel approaches the police and says he wants to make a deal. He agrees to tell the cops anything they want to know in return for a reduced charge. He subsequently tells the police the following information: "Danny and I brought the drugs up from Mexico and we planned to sell them to junior high school students." Before trial Weasel breaks out of jail and is nowhere to be found. At trial, the prosecutor calls the policeman to testify as to what Weasel told him. Danny objects to the officer's testimony as hearsay. How should the judge rule? A. Exclude the statement because the unavailability of the declarant has not been established. B. Admit the statement as a past recollection of the police officer. C. Exclude the statement because the declarant's motive for making the statement shows that it is not against his interest. D. Admit the statement as a statement of an opposing party's coconspirator.

C The correct answer is C. This is similar to the Williamson case. Because Weasel is trying to gain favor with the police in an attempt to reduce his own liability, his motivation is suspect and the relevant portions of the statement as a whole are not against his own interests under 804(b)(3). Answer choice A is wrong because under FRE 804(a) the declarant's unavailability has been established. Answer choice B is wrong because this is not a correct application of the 803(5) exception, as it is a not a recorded recollection and it is a statement of Weasel rather than the officer. Answer choice D is wrong because, even if there was evidence by a preponderance of a conspiracy between Danny and Weasel, the statement was not made during or in furtherance of the conspiracy.

In a 2018 trial of a tort action, Plaintiff Bob testifies he looked both ways before stepping into the street where he was struck by Defendant Dan's car. On cross examination, Dan's lawyer wants to ask Bob "Weren't you convicted in December 2016 of Felony Marijuana Possession, for which you received the maximum allowable sentence of one year's incarceration?" Bob's lawyer objects that this is inadmissible. How should the judge rule? A. Overrule the objection, as possession of illegal drugs is an inherently dishonest act. B. Overrule the objection, as Bob received a year's incarceration. C. Sustain the objection, as Bob could not have received a sufficiently severe punishment and possession of illegal drugs is not an inherently dishonest act. D. Sustain the objection, as this is a civil case.

C The correct answer is C. This prior conviction is inadmissible because it does not satisfy either FRE 609(a)(1) or 609(a)(2). It does not satisfy FRE 609(a)(1) the maximum punishment (which Bob received) was only one year, so he could not have received a sentence in excess of one year. Establishing the elements of marijuana possession in no way require proving a dishonest act or false statement, thus it does not satisfy FRE 609(a)(2). That this is a civil case is not relevant, because the crime did not carry a possible punishment in excess of one year.

Defendant is charged with the crime of insurance fraud (filing a false insurance claim with intent to deceive an insurer for payment) by allegedly reporting a staged accident (one that actually did not occur) in order to collect insurance benefits. As proof that she is not the type of person who would commit fraud, Defendant offers her minister, who testifies that he believes Defendant to be an honest person. The prosecution now offers rebuttal testimony by Defendant's neighbor that, among her neighbors, Defendant is considered dishonest. Defendant objects this is inadmissible character evidence. What should the judge do? A. Exclude the testimony because Defendant's evidence was in the form of opinion evidence, so reputation evidence is not in the proper form for rebuttal. B. Exclude the testimony because this character evidence is not in the form of specific acts. C. Admit the testimony because this is a fraud case so honesty is an essential element of the claim or defense. D. Admit the testimony because Defendant has introduced evidence of a pertinent trait of character.

D

Jane sues American Motors, which manufactured her car, claiming breach of the implied warranty of merchantability (that the car is suitable for its intended use) because the car's engine failed after two years and 20,000 miles. American Motors defends that the engine failed because Jane did not follow its recommendation to change the oil every six months or 5,000 miles. Jane offers testimony by a mechanic that he once changed oil in the car, six months after she bought it, when it had 5,000 miles on the odometer. American Motors objects that this testimony is not relevant. With respect only to the issue of relevance, how should the judge rule? A. The evidence is not relevant without proof that Jane followed the same schedule of changing the oil while she owned the car. B. The evidence is not relevant because Jane's claim is that as manufactured the car did not meet the basic requirements of a merchantable vehicle, without regard to what happened after purchase. C. The evidence is only conditionally relevant, subject to the admission later of proof that Jane followed the same schedule of changing the oil while she owned the car, otherwise it is not relevant. D. The evidence is relevant because it makes more likely that Jane timely changed the oil while she owned the car.

D

John "Maserati" Smith is indicted in Boston, MA for running an interstate car theft ring over the past three years, and he is arrested at the Miami (Florida) International Airport, as he is boarding a Boston-bound plane. The flight to Boston had a stop in Washington, D.C. on the way to Boston. At trial, the government seeks to introduce evidence that "when arrested, the defendant was at an international airport boarding a plane headed for Washington, D.C." as evidence of flight. The defendant objects. Which best describes the court's most likely decision? A. The judge should exclude the evidence because its probative value is substantially outweighed by the risk of unfair prejudice. B. The judge should admit the evidence because it is possible that the defendant was planning to leave the plane in Washington, D.C. C. The judge should admit the evidence subject to proof that the defendant's flight was actually motivated by a consciousness of guilt concerning the car theft ring. D. The judge should exclude the evidence because its probative value is substantially outweighed by the risk of misleading the jury.

D

Pam is injured when her "Vegomatic" slicer/dicer/spiralizer cuts off the tip of her finger, and she sues Vegomatic for failing to adequately warn users of the danger to their fingers. Shortly after her injury, Vegomatic places this warning at the top of its instructions: "Warning! Vegomatic blades are sharp! Keep fingers away from the blades!" At trial, Vegomatic's Chief of Product Engineering testifies, "Nothing could be done to make the Vegomatic a safer household appliance." Pam seeks to introduce Vegomatic's warning. Vegomatic objects. Which best describes how the court will likely rule? A. Exclude evidence of the warning, because it was a subsequent remedial measure. B. Admit evidence of the warning, because it was made before the testimony was given. C. Exclude evidence of the warning, because this is a products liability case. D. Admit evidence of the warning, because feasibility of precautionary measures is disputed.

D

Police respond to a 911 call of a suspicious person in a black coat near an elementary school who approached several children outside the school. Arriving at the school one minutes after the call, the police ask the principal for a description of the suspicious person, and the principal says, "he was short, maybe 5 feet, with a black ¾ length coat and a moustache. He was just here." The police rapidly search but find no one fitting this description. The following week a man exactly fitting this description is arrested near the school while trying to lure a child into a car. The man is charged with attempted child abduction, but by the time of trial the principal has died. The prosecution offers the principal's description from the week before the defendant's arrest as evidence of his plan. The defendant objects that the principal's statement is inadmissible hearsay. Which best describes what the judge will likely do? A. Admit the statement as a statement of an intent to engage in future conduct. B. Admit the statement as a statement identifying the defendant as someone the declarant perceived earlier. C. Admit the statement as a recorded recollection. D. Admit the statement as a present sense impression.

D Answer D is correct. The statement is admissible as a present sense impression. Fed. R. Evid. 803(1). The statement is made shortly after the event (the police come in three minutes and the principal says he was "just here"), and it is describing the event of the person's presence.

John is charged with robbery of a convenience store, and the clerk testifies that just before the robbery, John bought a pack of cigarettes that the clerk rang up on the register. As the clerk turned back with the change, John pointed a gun at him and said, "Give me the money and you won't get hurt!" After the clerk emptied the cash drawer into a bag and gave it to John, John said "get on the floor, and don't move." After the clerk heard John leave, he dialed 911 and reported the robbery. On the back of a candy wrapper he wrote, "Give me money," "5 feet 10 or 6 feet," "red hat," "silver gun with black handle" When the police arrived three minutes later, he gave a description of John and the wrapper with his notes, saying "I've worked here ten years and that's never happened." At trial a year later, the clerk cannot recall whether John was wearing a hat during the robbery or the color of the gun. The prosecutor asks if his memory is exhausted about these details. The clerk testifies that it is, but he thinks these details were on the notes he made at the time on a candy wrapper which he gave the police. When the prosecutor shows the clerk the candy wrapper, the clerk identifies it as the one he gave the police. The prosecutor moves to admit the candy wrapper with the notes into evidence to show the robber wore a hat and the color of the gun. John objects that the notes on the candy wrapper are hearsay. The prosecutor argues the notes are admissible as a record of a regularly conducted activity. How should the judge rule? A. Admit the notes as a record of a regularly conducted activity, because the record was made near the time of the event by someone with knowledge. B. Exclude the notes as a record of a regularly conducted activity, because the record was not one of an act, event, condition, opinion, or diagnosis. C. Admit the notes as a record of a regularly conducted activity, because the record was made while it was fresh in the clerk's mind. D. Exclude the notes as a record of a regularly conducted activity, because making the record was not a regular practice of that activity.

D Answer choice D is correct. The notes are not admissible as a record of a regularly conducted activity because, although they were made at or near the time of the event (the robbery) by one with knowledge (the clerk), it was not the regular practice of the activity to make this record. [Fed. R. Evid. 803(6)(C)] It was not the regular practice of the store clerk to make notes of the description of a robber. So answer choice A is wrong. The convenience store qualified as a business, and the notes were a record made near the time of the event (i.e., the robbery) by one with knowledge. It is even possible that the "activity" could be running the register or operating the counter of the store, in which case the record would be kept in the course of a regularly conducted activity of the store (running the counter). So answer choice B is wrong. Making this particular record, however, was not a regular practice of operating the counter. There had never been a robbery in ten years, so it hardly a regular practice for the clerk to record the descriptions of robbers. So answer choice C is wrong.

Mary is charged with embezzlement from her employer by pocketing money from the cash drawer, and the prosecution calls a co-worker, Sharon, to testify she often saw Mary "rearranging" the cash drawer for no apparent reason. Mary seeks to call Tom, a co-worker of both Sharon and Mary, to testify that Sharon often stole office supplies from their workplace. The prosecution objects. How should the judge rule? A. This is admissible other acts evidence offered to show Sharon's character for dishonesty. B. This is admissible character evidence because the charge is embezzlement. C. This is admissible character evidence because, by testifying, Sharon's character for honesty is at issue. D. This is inadmissible character evidence offered as impeachment because it is in an improper form.

D Correct answer is D. This is character evidence offered as impeachment but it is inadmissible because it is in an improper form. This is character evidence because it invites the fact-finder to infer something about Sharon's character (that she is untruthful) from other instances of behavior (stealing office supplies). It is evidence of specific acts (stealing office supplies) to show a character for dishonesty. This is not permissible; character evidence concerning the character of a witness for truthfulness or untruthfulness must be by reputation or opinion, and extrinsic evidence is not permissible. A. is wrong because other acts that are probative of untruthfulness (like these) may be asked about only on cross examination of the witness; they may not be introduced through extrinsic evidence. B. is wrong because the nature of the charge is irrelevant. C. is wrong because while Sharon's character for honesty is at issue because she has testified, it may only be impeached through reputation or opinion evidence. These are specific acts, which are inadmissible through extrinsic evidence. Tom's testimony is extrinsic evidence. These acts may only be asked of the witness (Sharon) on cross examination. Fed. R. Evid. 608(b)(1).

In a robbery case, the prosecution seeks to offer the Defendant's statement in a phone call to his mother, overheard by a police officer at the station after his arrest, that "he wasn't going to be home for a few years," as evidence of his consciousness of guilt. Defendant objects that this is hearsay. Which of the following best describes what the judge should do? A. Sustain the objection, because the Defendant is not asserting anything related to the robber. B. Sustain the objection, unless the prosecution also has the Defendant's mother testify about what she said that prompted this statement. C. Overrule the objection, as this a statement Defendant manifested he believed to be true. D. Overrule the objection, as this is a statement made by a party.

D The correct answer is D. This is a statement made by a party opponent (i.e., the Defendant is the party opponent to the government), thus it is admissible under FRE 801(d)(2)(A). Answer A is incorrect because Defendant is asserting what he says, namely that he does not believe that he will be home for a few years, and whether it is related to the robbery is a question of relevance. In this context, it could show consciousness of guilt. Answer B is incorrect. The mother is not needed to admit the Defendant's part of the conversation, although there may be a FRE 403 argument that the evidence is misleading without her testimony because it misrepresents the meaning of the conversation. Answer C is incorrect. It is not a statement in which Defendant manifested a belief in the truth of, or that the defendant adopted. It is a statement the Defendant, who is the party opponent, made.


Conjuntos de estudio relacionados

Prep Quiz #9 GI/Peptic Ulcer Disease

View Set

NUR379 Evolve: Health Promotion and Safety - Health Promotion for Infants, MCH Sherpath Health Promotion Toddlers, Peds: Key Concepts and Theories of Growth and Development

View Set

4.14.F - Test: Module Four Business STUSY SET

View Set

История Казахстана 1-650

View Set